解析力学/ラグランジアン のバックアップ差分(No.18)

更新


  • 追加された行はこの色です。
  • 削除された行はこの色です。
>>> [[解析力学]] へ戻る

&katex();

* 目次 [#z557608a]

#contents

* ラグランジアンの定義 [#oa193382]

力学系の運動エネルギーを $T$、ポテンシャルエネルギーを $U$ とするとき、
ラグランジアン $L$ を、

$$
L=T-U
$$

として定義する。

** 例:重力を及ぼしあう2つの星 [#fbd79e12]

互いに重力を及ぼしあう2つの星の運動を考える。片方の星の質量を $M$、座標を $X,Y,Z$、もう一方の星の質量を $m$、座標を $x,y,z$、万有引力定数を $G$ とすれば、運動エネルギー、ポテンシャルエネルギーは

$$
T=\frac12 M\bigg[\bigg(\frac{dX}{dt}\bigg)^2+\bigg(\frac{dY}{dt}\bigg)^2+\bigg(\frac{dZ}{dt}\bigg)^2\bigg]+
\frac12 m\bigg[\bigg(\frac{dx}{dt}\bigg)^2+\bigg(\frac{dy}{dt}\bigg)^2+\bigg(\frac{dz}{dt}\bigg)^2\bigg]
$$
$$
U=-G\frac{Mm}{\sqrt{(X-x)^2+(Y-y)^2+(Z-z)^2}}
$$

となるから、ラグランジアンは

$$
\begin{aligned}
L=&\frac12 M\bigg[\bigg(\frac{dX}{dt}\bigg)^2+\bigg(\frac{dY}{dt}\bigg)^2+\bigg(\frac{dZ}{dt}\bigg)^2\bigg]+\frac12 m\bigg[\bigg(\frac{dx}{dt}\bigg)^2+\bigg(\frac{dy}{dt}\bigg)^2+\bigg(\frac{dz}{dt}\bigg)^2\bigg]\rule[-6mm]{0mm}{0mm}\\
&-\bigg[-G\frac{Mm}{\sqrt{(X-x)^2+(Y-y)^2+(Z-z)^2}}\bigg]
\end{aligned}
$$

と書ける。
* 力学系の座標と速度 [#a3b96963]

以下では力学系を記述するための座標を $q_1,q_2,\dots,q_n$ で表すことにする。

>質点1個の運動なら $n=3$ で、$q_1=x,q_2=y,q_3=z$ と考えればいい。

>質点2個の運動なら $n=6$ で、$q_1=x,q_2=y,q_3=z,q_4=X,q_5=Y,q_6=Z$ と考えればいい。

>質点1個が原点を中心とする半径 $r$ の球面上を動くなら、$n=2$ として $q_1=\theta,q_2=\phi$ と取れば良い。

また、時間による全微分を $\displaystyle\dot q_k=\frac{dq_k}{dt}$、 $\displaystyle\ddot q_k=\frac{d^2q_k}{dt^2}$ のように変数の上に点を付けて表すことにする。

さしあたり運動エネルギーは $q_1,q_2,\dots,q_n$ および $\dot q_1,\dot q_2,\dots,\dot q_n$ の関数、ポテンシャルエネルギーは $q_1,q_2,\dots,q_n,\dot q_1,\dot q_2,\dots,\dot q_n$ と $t$ の関数であるとする。すると、

$$
L(q_1,q_2,\dots,q_n,\dot q_1,\dot q_2,\dots,\dot q_n,t)=T(q_1,q_2,\dots,q_n,\dot q_1,\dot q_2,\dots,\dot q_n)-U(q_1,q_2,\dots,q_n,\dot q_1,\dot q_2,\dots,\dot q_n,t)
$$

の形になる。

カーテシアン座標系($x,y,z$座標系)を思い浮かべていると運動エネルギーに位置座標 $q_1,q_2,\dots,q_n$ が含まれるのはおかしいと感じるかもしれないが、例えば極座標で表すと速度の表式に $r$ や $\sin\theta$ が現れることを思い出そう。

ポテンシャルに $t$ を含んでもよいとしているのは、強制振動のように系外から力がかかりポテンシャルが変化する場合を表せるようにするためである。

ポテンシャルがいわゆる保存力を表す場合、ポテンシャルは速度 $\dot q_i$ に依存しないが、$U$ を $\dot q_i$ に依存する形にすることでローレンツ力のような「保存力ではない力」を取り扱うことができるようになる。

さらに広範な問題を扱うには $L=T-U$ の形以外のラグランジアンを考えることもあるため、
今後の話はできるかぎりラグランジアンの形に何ら制約を課さずに進め、
そうでない場合には特に断ることにする。

* 最小作用の原理 [#maf140c9]

時刻 $t_1$ における座標値 $q_k(t_1)$  と、時刻 $t_2$ における座標値 $q_k(t_2)$ が与えられたとすると、
その間での系の運動 $q_k(t)$ は「作用」と呼ばれる値(ラグランジアンの積分値として与えられる)

$$
S=\int_{t_1}^{t_2}L\ dt
$$

を最小にするようなものとなる。というのが最小作用の原理である。

何を言っているかというと、各座標の最初と最後の値が決まってもその間で座標が時間と共にどのように変化するかはいろんな可能性が考えられる。しかし、そのうち実際に起きる運動は $S$ を最小とするものになる、ということだ。

さらに言い換えると、一般に運動方程式は時間に対する2階の微分方程式であるから、その一般解は1つの座標につき2つの自由パラメータを持つ。始点と終点という2つのパラメータを指定すると運動は1つに定まる(初期座標と初期速度を与えるのと同じ)。その定まった軌道が $S$ を最小にするものだ、ということだ。

え、そんな話は聞いたことがないんだけど、と思うだろうけど実際こういう原理が存在するのだから仕方がない。最小作用の原理がこれまで学んできたニュートン方程式と矛盾しないことを以下で見る。

* ラグランジュの運動方程式 [#x337268d]

「作用を最小化する」という条件を微分方程式の形に直したものをラグランジュの運動方程式と呼ぶ。

** 汎関数 [#kdc0fbdb]

$S$ は関数 $q_k(t)$ を1つ決めると1つ値が決まるという意味で「汎関数」と呼ばれる。$S$ が $q_k(t)$ の汎関数であることを強調する際には

$$S=S\big[q_k(t)\big]$$

などと書く。

ある座標の軌跡 $q_k(t)$ を変化させて $q_k(t)+\delta q_k(t)$ としたとする。$q_k(t)$ が最小作用を与えるという意味は、どのように $\delta q_k(t)$ を選んだとしても $S$ は元の値より増加する、ということと同義である。

$$S[q_k(t)+\delta q_k(t)]>S[q_k(t)]$$

ただし始点と終点は決まっているので $\delta q_k(t_1)=\delta q_k(t_2)=0$ とする。

$\delta q_k(t)$ が小さいときを扱うことにして、$S$ を $q_k(t)$ で「微分」することを考える。

** 汎関数微分 [#u34af725]

$S$ の $q_k$ による「汎関数微分 $\frac{\delta S}{\delta q_k}$」は、

$$
\begin{aligned}
\delta S&=S\big[q_k(t)+\delta q_k(t)\big]-S\big[q_k(t)\big]\\
&\sim\int_{t_1}^{t_2}\frac{\delta S}{\delta q_k}\delta q_k(t)\,dt
\end{aligned}
$$

として定義される。

普通の関数 $f(x)$ の微分が

$$
\begin{aligned}
\Delta f&=f(x+\Delta x)-f(x)\\
&\sim\frac{df}{dx}\Delta x
\end{aligned}
$$

であるのと比べると、汎関数微分の定義になぜ積分が現れるのか疑問に思うかもしれない。

その疑問は、多変数関数 $f(x_1,x_2,\dots,x_n)$ の偏微分が、

$$
\begin{aligned}
\Delta f&=f(x_1+\Delta x_1,\ x_2+\Delta x_2,\ \dots\ ,x_n+\Delta x_n)-f(x_1,x_2,\dots,x_n)\\
&\sim\sum_{k=1}^n\frac{\partial f}{\partial x_k}\Delta x_k
\end{aligned}
$$

であることと比べると解決するだろう。

汎関数 $S$ は $t_1$ から $t_2$ までのすべての時刻に対する $q_k(t)$ の値を引数とする超多変数関数であると見做せるため(個数の引数は連続無限)、それら個々の値の変化に、その値に対する微分値 $\frac{\delta S}{\delta q_k}$ を掛けて足したものが全体としての変化となるわけだ。

ということで、当然だが汎関数微分 $\frac{\delta S}{\delta q_k}$ は時刻の関数となる。

** 最小点においては汎関数微分がゼロとなる [#ifcb74cb]

$f(x)$ の極大点・極小点において $\frac{df}{dx}=0$ が成り立つのと同様に、最小作用を与える $q_k(t)$ に対して、

$$\frac{\delta S}{\delta q_k}=0$$

が成り立つ必要がある。

実際に汎関数微分を計算して $q_k$ に対する条件を求めよう。その際、$q_k(t)$ を変化させると $\dot q_k(t)$ も変化することを加味する必要がある。さらに $\delta q_k(t_1)=\delta q_k(t_2)=0$ を用いた部分積分を適用すると、

$$\begin{aligned}
\delta S
&=\int_{t_1}^{t_2} \Big[L(\dots,q_k(t)+\delta q_k(t),\dots,\dot q_k(t)+\delta \dot q_k(t),\dots)-L(\dots,q_k(t),\dots,\dot q_k(t),\dots)\Big]\,dt\\
&=\int_{t_1}^{t_2}\bigg[\frac{\partial L}{\partial q_k}\delta q_k+\frac{\partial L}{\partial \dot q_k}\delta \dot q_k\bigg]\,dt\\
&=\cancel{\bigg[\frac{\partial L}{\partial \dot q_k}\delta  q_k\bigg]_{t_1}^{t_2}}+\int_{t_1}^{t_2}\bigg[\frac{\partial L}{\partial q_k}\delta q_k-\frac{d}{dt}\bigg\{\frac{\partial L}{\partial \dot q_k}\bigg\}\delta q_k\bigg]\,dt\\
&=\int_{t_1}^{t_2}\underbrace{\bigg[\frac{\partial L}{\partial q_k}-\frac{d}{dt}\bigg\{\frac{\partial L}{\partial \dot q_k}\bigg\}\bigg]}_{=\,\frac{\delta S}{\delta q_k}}\delta q_k\,dt=0\\
\end{aligned}$$

を得る。これが任意の $\delta q_k(t)$ に対して成り立つためには汎関数微分 $\frac{\delta S}{\delta q_k}$ をゼロと置く必要があり、次の方程式が得られる。

$$
\frac{d}{dt}\bigg\{\frac{\partial L}{\partial \dot q_k}\bigg\}-\frac{\partial L}{\partial q_k}=0
$$

この方程式はラグランジュの運動方程式と呼ばれ、任意の $t$ および $k$ に対してこの式が成り立つことが「停留作用」の法則の微分形を与える。この方程式に現れる $\frac{\partial L}{\partial \dot q_k}$ や $\frac{\partial L}{\partial q_k}$ は、$\dot q_k$ と $q_k$ とを「独立の変数」とみなして偏微分したものであることに注意せよ。

本来ラグランジュの運動方程式自体は作用が最小となることを保証しない。作用が停留値を取ることのみを与える。ラグランジュの運動方程式を解いて得られた軌道の近傍で小刻みに振動するような軌道を考えれば、ポテンシャルエネルギーをほとんど変えないまま運動エネルギーを増加させることがいつでも可能である。したがって、求まる停留値が極大値である可能性はない。ただしそれが鞍点ではないのか、あるいはもし極小値であったとしてもそれが真の最小値であるのか、についてはここまでの議論からは判然としない。

** ニュートン方程式が導かれる [#a6468f36]

ポテンシャルエネルギー $U(x,y,z,t)$ 中で運動する質量 $m$ を持つ質点の運動は、ニュートン方程式

$$
\begin{cases}
m\ddot x=-\frac{\partial}{\partial x} U\\
m\ddot y=-\frac{\partial}{\partial y} U\\
m\ddot z=-\frac{\partial}{\partial z} U\\
\end{cases}
$$

で記述される。(力は $\bm f=-\bm \nabla U$ で与えられることを思い出せ)

一方、この系の運動エネルギーは $T=\frac{1}{2}m(\dot x^2+\dot y^2+\dot z^2)$ であるから、

$$
L=T-U=\frac{1}{2}m(\dot x^2+\dot y^2+\dot z^2)-U(x,y,z)
$$

である。

ラグランジュの運動方程式は、

$$
\begin{cases}
\displaystyle\frac{d}{dt}\bigg\{\frac{\partial L}{\partial \dot x}\bigg\}=\frac{\partial L}{\partial x}
\ \ \to\ \ \frac{d}{dt}(m\dot x)=-\frac{\partial}{\partial x}U\\
\displaystyle\frac{d}{dt}\bigg\{\frac{\partial L}{\partial \dot y}\bigg\}=\frac{\partial L}{\partial y}
\ \ \to\ \ \frac{d}{dt}(m\dot y)=-\frac{\partial}{\partial y}U\\
\displaystyle\frac{d}{dt}\bigg\{\frac{\partial L}{\partial \dot z}\bigg\}=\frac{\partial L}{\partial z}
\ \ \to\ \ \frac{d}{dt}(m\dot z)=-\frac{\partial}{\partial z}U
\end{cases}
$$

となって、確かにニュートン方程式と一致する。

実はラグランジアンや作用を適切に定義することにより、
ニュートン方程式だけでなくマクスウェル方程式や一般相対論などについても
最小作用の形で定式化できることが知られている。

** 一般化運動量(正準運動量・力学的運動量) [#o9459a42]

上で見たとおりラグランジュの運動方程式がニュートン方程式を与える際には

$$
\frac{d}{dt}\bigg\{\underbrace{\frac{\partial L}{\partial \dot q_k}}_\text{運動量}\bigg\}=\underbrace{\frac{\partial L}{\partial q_k}}_\text{力}
$$

の形になり、力により運動量が時間変化する、という式になった。

そこで一般の座標 $q_k$ に対しても、

$$
p_k=\frac{\partial L}{\partial \dot q_k}
$$

を $q_k$ に対応する運動量であると呼ぶことにする。例えば $q_k$ が極座標の $\theta$ であれば、この値はそもそも通常の運動量と同じ次元すらもたないのだが、それでもこれを $p_\theta$ と書いて運動量として扱う。
このような「運動量」は通常の運動量と区別するために一般化運動量あるいは正準運動量や力学的運動量と呼ばれる。

このときラグランジュの運動方程式は

$$
\dot p_k=\underbrace{\frac{\partial L}{\partial q_k}}_\text{一般化力}
$$

を表すことになる。一般化運動量を変化させる右辺は一般化力とも呼ばれる。

** ラグランジアンの全微分 [#r4e5d014]

ラグランジアンの全微分は

$$
\begin{aligned}
dL
&=\sum_{k=1}^n\left[\frac{\partial L}{\partial \dot q_k}d\dot q_k+\frac{\partial L}{\partial q_k}d q_k\right]+\frac{\partial L}{\partial t}dt\\
&=\sum_{k=1}^n\big(p_k\,d\dot q_k+\dot p_k\,d q_k\big)+\frac{\partial L}{\partial t}dt\\
\end{aligned}
$$

のように対称性の良い形に表せることになる。

* 例題 [#qd0685db]

ラグランジュ方程式がニュートン方程式を与えるだけであるなら、なぜ新たにラグランジュ方程式を考える必要があるのだろう。その一端を見るために2つ例題を見てみよう。

** $x$ 軸上の点電荷 [#ic5a4aaf]

$x$ 軸上に限り摩擦なく自由に動ける点電荷 $q$ (質量 $m$) が
$(0,1)$ に固定された点電荷 $-q$ からのクーロン力と、運動を $x$ 軸上に限るための拘束力のみを受けて運動するとして、点電荷の $x$ 座標の時間変化を表す運動方程式を求めたい。ただし系の誘電率を $\epsilon_0$ とする。

これまでと同様に普通にニュートン方程式を立てるなら、

電荷間の引力は $\displaystyle-\frac{1}{4\pi\epsilon_0}\frac{e^2}{x^2+1}$

その $x$ 軸に沿った成分は $\displaystyle-\frac{1}{4\pi\epsilon_0}\frac{e^2}{x^2+1}\frac{x}{\sqrt{x^2+1}}$

したがって、次の運動方程式を得る。

$$
m\ddot x=-\frac{1}{4\pi\epsilon_0}\frac{e^2}{x^2+1}\frac{x}{\sqrt{x^2+1}}
$$

一方、ラグランジアンを使うと、

運動エネルギー $\displaystyle T=\frac{1}{2}m\dot x^2$

ポテンシャルエネルギー $\displaystyle U=\frac{1}{4\pi\epsilon_0}\frac{e^2}{\sqrt{x^2+1}}$

ラグランジアン $L=T-U$

$$
L=\frac{1}{2}m\dot x^2-\frac{1}{4\pi\epsilon_0}\frac{e^2}{\sqrt{x^2+1}}
$$

ラグランジュの運動方程式

$$
\frac{d}{dt}\left(\frac{\partial L}{\partial \dot x}\right)-\frac{\partial L}{\partial x}=0
$$

$$
m\ddot x=-\frac{e^2}{4\pi\epsilon_0}\frac{x}{(x^2+1)^{3/2}}
$$

として同じ式が得られる。

$x$ に対応する運動量は、

$$
p_x=\frac{\partial L}{\partial \dot x}=m\dot x
$$

となって、ちゃんと我々のなじみのある形が出てきた。

** 振り子の運動 [#sebbcf9b]

長さ $r$、重りの質量 $m$ の振り子について、その振れ角を $\theta$ として、$\theta$ に対する運動方程式を求めたい。

これまで通りにやるなら重力加速度を $g$ として、

重力 $mg$

その円弧に沿った成分 $-mg\sin\theta$

円弧に沿った加速度 $r\ddot\theta$

より、運動方程式は

$$
mr\ddot\theta=-mg\sin\theta
$$

すなわち、次の運動方程式が得られる。

$$
\ddot\theta=-\frac gr\sin\theta
$$

一方、ラグランジアンを用いる場合には、

重りの速度 $r\dot\theta$

高さは $r-r\cos\theta$ と表せるから、

運動エネルギー $T=\frac12 mr^2\dot\theta^2$

ポテンシャルエネルギー $U=mgr(1-\cos\theta)$

ラグランジアン $L=\frac12 mr^2\dot\theta^2-mgr(1-\cos\theta)$

$\theta$ に対する運動方程式は、

$$
\frac{d}{dt}\left\{\frac{\partial L}{\partial\dot\theta}\right\}=\frac{\partial L}{\partial\theta}
$$

$$
mr^2\ddot\theta=-mgr\sin\theta
$$

すなわち、

$$
\ddot\theta=-\frac{g}{r}\sin\theta
$$

を得る。

$\theta$ に対応する一般化運動量は、

$$
p_\theta=\frac{\partial L}{\partial \dot \theta}=mr^2\dot\theta
$$

となる。

* ラグランジュ力学のメリット [#z9cb4f3d]

ラグランジアンを用いた方法では力の働く方向などを考える必要がなく、
ラグランジアンさえ求めてしまえばそれを形式的に微分するのみで
運動方程式が得られる。

そして、運動を記述するのが $x,y,z$ 座標であろうが、$r,\theta,\phi$ であろうが、
基本方程式を同じ形($\dot p_k=\partial L/\partial q_k$)で記述できることが非常に重要である。

この性質を指して、運動方程式が座標変換に対して共変である、という。

** ラグランジュの運動方程式の共変性 [#kf999ae0]

ラグランジュの運動方程式の共変性は、その元となった最小作用の法則が座標の取り方に寄らない形式になっているため改めて確かめるまでもなく当然なりたつのであるが、ここでは練習も兼ねて明示的に示しておく。

座標系 $q_1,q_2,\dots,q_n$ から別の座標系 $Q_1,Q_2,\dots,Q_n$ への変数変換が、

$$
Q_i=Q_i(q_1,q_2,\dots,q_n)\ \ \ \ \ (i=1,2,\dots,n)
$$

で与えられるとする。またこの変換には逆変換が存在して、

$$
q_i=q_i(Q_1,Q_2,\dots,Q_n)\ \ \ \ \ (i=1,2,\dots,n)
$$

とする。

この座標変換は $x,y,z$ から $r,\theta,\phi$ のような幾何学的な座標変換であっても良いし、
2つの粒子の座標を重心座標と相対座標とに分けたり、あるいは
バネで連結された多数の質点の個々の位置座標から基準振動の振幅と位相へ変換するなど、どんな変換でも構わない。

座標 $q_1,q_2,\dots,q_n$ を決めれば座標 $Q_1,Q_2,\dots,Q_n$ を求められ、
逆に座標 $Q_1,Q_2,\dots,Q_n$ を決めれば座標 $q_1,q_2,\dots,q_n$ を求められる、
ということだけが座標変換の条件である。

このとき、すべての $i=1,2,\dots,n$ に対して、

$$
\frac{d}{dt}\left\{\frac{\partial L}{\partial \dot q_i}\right\}-\frac{\partial L}{\partial q_i}=0
$$

が成り立つことを仮定すると、すべての $i=1,2,\dots,n$ に対して、

$$
\frac{d}{dt}\left\{\frac{\partial L}{\partial \dot Q_i}\right\}-\frac{\partial L}{\partial Q_i}=0
$$

が成り立つことが以下のように導ける。(この話は $q_i$ と $Q_i$ に対して対称なので、当然逆も成り立つ)

まず、上記の変数変換では $Q_j$ は $q_1,q_2,\dots,q_n$ のみで決まり $\dot q_i$ に依存しないため、

$$
\frac{\partial Q_i}{\partial \dot q_j}=0
$$

また、

$$
\dot Q_i=\frac d{dt}Q_i(q_1,q_2,\dots,q_n)=\sum_{j=1}^n\frac{\partial Q_i}{\partial q_j}\dot q_j
$$

より、

$$
\frac{\partial \dot Q_i}{\partial \dot q_j}=\frac{\partial Q_i}{\partial q_j}
$$

であるが、一方、一般に $\frac{\partial Q_i}{\partial q_j}$ は $q_1,q_2,\dots,q_n$ の関数となるため、

$$
\frac{\partial\dot Q_i}{\partial q_j}=\sum_{k=1}^n\frac{\partial^2 Q_i}{\partial q_j\partial q_k}\dot q_k
$$

である。同様にして、

$$
\frac{\partial q_i}{\partial \dot Q_j}=0,
\ \ \ \ \ \frac{\partial \dot q_i}{\partial \dot Q_j}=\frac{\partial q_i}{\partial Q_j},
\ \ \ \ \ \frac{\partial\dot q_i}{\partial Q_j}=\sum_{k=1}^n\frac{\partial^2 q_i}{\partial Q_j\partial Q_k}\dot Q_k
$$

である。これらを用いて $L$ の $Q_i,\dot Q_i$ に対する偏微分を求めれば、

$$
\frac{\partial L}{\partial Q_i}
=\sum_{j=1}^n\frac{\partial q_j}{\partial Q_i}\frac{\partial L}{\partial q_j}+\sum_{j=1}^n\frac{\partial\dot q_j}{\partial Q_i}\frac{\partial L}{\partial\dot q_j}
$$

$$
\begin{aligned}
\frac{\partial L}{\partial\dot Q_i}
&=\sum_{j=1}^n\cancel\frac{\partial q_j}{\partial\dot Q_i}\frac{\partial L}{\partial q_j}+\sum_{j=1}^n\frac{\partial\dot q_j}{\partial\dot Q_i}\frac{\partial L}{\partial\dot q_j}\\
&=\sum_{j=1}^n\frac{\partial q_j}{\partial Q_i}\frac{\partial L}{\partial\dot q_j}\\
\end{aligned}
$$

上式を代入すると、

$$
\begin{aligned}
\frac{d}{dt}\left\{\frac{\partial L}{\partial \dot Q_i}\right\}-\frac{\partial L}{\partial Q_i}
&=\sum_{j=1}^n\left[\frac{d}{dt}\left\{\frac{\partial q_j}{\partial Q_i}\frac{\partial L}{\partial \dot q_j}\right\}-\frac{\partial q_j}{\partial Q_i}\frac{\partial L}{\partial q_j}-\frac{\partial\dot q_j}{\partial Q_i}\frac{\partial L}{\partial\dot q_j}\right]\\
&=\sum_{j=1}^n\bigg[\frac{d}{dt}\left\{\frac{\partial q_j}{\partial Q_i}\right\}\frac{\partial L}{\partial \dot q_j}+\underbrace{\frac{\partial q_j}{\partial Q_i}\frac{d}{dt}\left\{\frac{\partial L}{\partial \dot q_j}\right\}-\frac{\partial q_j}{\partial Q_i}\frac{\partial L}{\partial q_j}}_{q_j,\dot q_j\text{に対する運動方程式よりゼロ}}-\frac{\partial\dot q_j}{\partial Q_i}\frac{\partial L}{\partial\dot q_j}\bigg]\\
&=\sum_{j=1}^n\bigg[\bigg(\frac{\partial}{\partial Q_i}\frac{dq_j}{dt}\bigg)\frac{\partial L}{\partial \dot q_j}-\frac{\partial\dot q_j}{\partial Q_i}\frac{\partial L}{\partial\dot q_j}\bigg]=0\\
\end{aligned}
$$

最下行冒頭の等号では $Q_i$ による偏微分と時間による全微分を入れ替えた。

* ハミルトニアンの定義とエネルギーの保存 [#b9eedbd5]

任意の $L$ に対して $t$ から $t+\delta t$ の間の変化を求めると、

$$
\begin{aligned}
\delta L=\frac{dL}{dt}\delta t=& L(q_1(t+\delta t),q_2(t+\delta t),\dots,q_n(t+\delta t),\dot q_1(t+\delta t),\dot q_2(t+\delta t),\dots,\dot q_n(t+\delta t),t+\delta t)\\
&-L(q_1(t),q_2(t),\dots,q_n(t),\dot q_1(t),\dot q_2(t),\dots,\dot q_n(t),t)\\
=&\bigg[\sum_{i=1}^n\bigg(\frac{\partial L}{\partial q_i}\dot q_i+\frac{\partial L}{\partial\dot q_i}\ddot q_i\bigg)+\frac{\partial L}{\partial t}\bigg]\delta t\\
=&\bigg[\sum_{i=1}^n\bigg\{\frac{d}{dt}\bigg(\frac{\partial L}{\partial\dot q_i}\bigg)\dot q_i+\frac{\partial L}{\partial\dot q_i}\ddot q_i\bigg\}+\frac{\partial L}{\partial t}\bigg]\delta t\\
=&\bigg[\frac{d}{dt}\bigg(\sum_{i=1}^n\frac{\partial L}{\partial\dot q_i}\dot q_i\bigg)+\frac{\partial L}{\partial t}\bigg]\delta t\\
\end{aligned}
$$

すなわち、

$$
\frac{d}{dt}\bigg\{\sum_{i=1}^n\frac{\partial L}{\partial\dot q_i}\dot q_i-L\bigg\}=-\frac{\partial L}{\partial t}
$$

が成り立つ。そこで $\{\ \ \}$ の中味を

$$
H=\sum_{i=1}^n\frac{\partial L}{\partial\dot q_i}\dot q_i-L=\sum_{i=1}^n p_i\dot q_i-L
$$

と置いて、この値をハミルトニアンと呼ぶ。すると、

$$
\dot H=-\frac{\partial L}{\partial t}
$$

と書けるから、もし右辺がゼロ、すなわちラグランジアンが陽に時間に依存していなければ、
ハミルトニアンは時間に対する保存量となる。右辺がゼロでなければこれは単位時間当たりの
ハミルトニアンの減少量が $\frac{\partial L}{\partial t}$ で書けることを表す。

さて、この保存量は物理的には何に当たるのだろうか?

実は $L=T-U$ と書ける場合、ハミルトニアンは系の全エネルギーに相当する。

$L=T-U$ と書ける場合、ラグランジアンが陽に時間に依存するというのはポテンシャル $U$ が時間に依存する場合であり、この時、系外にポテンシャルを変化させている「誰か」が存在して、その誰かと系との間でエネルギーのやりとりが行われている場合に相当する。

逆にポテンシャルが時間に依存しないことは系を孤立系と見做せることと同義であり、そのとき系のエネルギーが保存する、というのが上記の結果である。

この「エネルギーの保存則」は多くの教科書で「ネーターの定理」の帰結として説明されているのだが、良い説明になっていないことも多いようだ。そこで別ページに、

- [[ラグランジアンの任意性>解析力学/ネーターの定理#mcce419c]]
- [[ラグランジアンの対称性>解析力学/ネーターの定理#jd0ed387]]
- [[ネーターの定理>解析力学/ネーターの定理#ic343e17]]
-- [[並進対称性 → 運動量保存>解析力学/ネーターの定理#j4e2b3f1]]
-- [[回転対称性 → 角運動量保存>解析力学/ネーターの定理#ia09e24a]]
- [[ネーターの定理とエネルギー保存>解析力学/ネーターの定理#pb954f92]]
-- [[例:強制振動 >解析力学/ネーターの定理#b9d982cd]]
-- [[例:粘性抵抗のある場合>解析力学/ネーターの定理#ec972d15]]

についてまとめた。

ネーターの定理は、ラグランジアンが何らかの座標変換に対して対称性を持つとき、それに対応する「保存則」が現れる、というものだ。並進対称性を持てば運動量保存則が、回転対称性を持てば角運動量保存則が現れる。これに対応して、時間に対する並進対称性がエネルギー保存則と深い関係を持つのであるが、その導出は他のネーターの定理とは少し趣が変ったものとなる。また、この保存則を正しく理解することで、外界との間でエネルギーのやりとりがある場合や、粘性抵抗が存在する場合のように、エネルギーが保存しない場合の「保存則」を導出ることが可能となる。

ハミルトニアンが確かに系の全エネルギーに相当することを上の例で試しておこう。

** $x$ 軸上の点電荷の運動 [#q4a02206]

$$
\begin{aligned}
H&=m\dot x\times \dot x-(\frac12m\dot x^2-U)\\
&=\frac12m\dot x^2+U\\
&=T+U
\end{aligned}
$$

** 振り子の運動 [#pd1847fa]

$$
\begin{aligned}
H&=mr^2\dot \theta\times \dot \theta-(\frac12mr^2\dot \theta^2-U)\\
&=\frac12mr^2\dot \theta^2+U\\
&=T+U\\
\end{aligned}
$$

* ポテンシャルとエネルギー [#b36859db]

$L=T-U$ と書ける場合、上のようにして求めたハミルトニアンは、

$$
H=\sum_{i=1}^n p_i\dot q_i-L=\sum_{i=1}^n p_i\dot q_i-T+U
$$

となるから、$\displaystyle\sum_{i=1}^n p_i\dot q_i=2T$ であれば

$$
H=T+U
$$

となり、期待通りハミルトニアンが運動エネルギーとポテンシャルエネルギーの和として表せる。

実際、ポテンシャルエネルギー $U$ が $\dot q_k$ を含まず、また運動エネルギー $T$ が $\dot q_k^2$ の線形結合で

$$
T=\sum_{i=1}^n \frac12m_i(q_1,q_2,\dots,q_n)\dot q_i^2
$$

の形に表されるとき(ここでの $m_i$ は一般に座標の関数であり質量そのものに限定しない。上の振り子の例の $T=\frac12mr^2\dot\theta^2$ であれば $mr^2$ に相当する)、上記の項は、

$$
\begin{aligned}
\sum_{i=1}^n p_i\dot q_i&=\sum_{i=1}^n \frac{\partial L}{\partial \dot q_i}\dot q_i\\
&=\sum_{i=1}^n \frac{\partial T}{\partial \dot q_i}\dot q_i\\
&=\sum_{i=1}^n m_i\dot q_i^2=2T\\
\end{aligned}
$$

となり、運動エネルギーの2倍 $2T$ に等しくなる。

カーテシアン座標系における質点の運動エネルギーはまさに上記の形で $m_i$ を質点の質量としたものに等しいし、剛体系で物体の回転を表す回転角を座標に取った場合にも、その回転エネルギーは上記の形となる。

さらに、後に見るように$\sum_{i=1}^n p_i\dot q_i$の値は座標系の取り方によらない、すなわち同じ系を異なる座標 $Q_i$ で表した場合にも $\sum_{i=1}^n P_i\dot Q_i=\sum_{i=1}^n p_i\dot q_i=2T$ となるから、もっと変った座標を取ったとしても、この値が運動エネルギーの2倍に等しいことは変らない。

一方で、ローレンツ力を含む場合のように $U$ が速度 $\dot q_i$ を含む場合などはそのようにうまく行かず、
「ハミルトニアンは $T+U$ と等しくならない」。

それでも $L=T-U$ と書けるときハミルトニアン $H$ は系の全エネルギーに等しいため((たぶん成り立つのだと思うのだけれど、ちゃんと証明するにはどうしたらいいのか、個人的に理解できてない))、
そのようなケースでラグランジアンに現れる「ポテンシャル」$U$ は、全エネルギーを計算するためには使えない、あくまで力を計算するために定義された便宜的なものとなる。ローレンツ力を扱う際の $U$ がその代表である。

まとめると、

- $L=T-U$ が成り立ち、$\displaystyle\bigg(\sum_{i=1}^n p_i\dot q_i\bigg)=2T$ であれば系のエネルギーは  $H=T+U$ と表せる。
- $L=T-U$ が成り立つものの、$U$ が速度を含むような場合には $\displaystyle\sum_{i=1}^n p_i\dot q_i\ne\ 2T$ となり、系のエネルギーは $H\ne T+U$ である。すなわちこの場合の $U$ は本来の意味でのポテンシャルではない。

これらのケースでは、

$$
\dot H=-\frac{\partial L}{\partial t}
$$

は時間当たりに系から失われるエネルギーが $\frac{\partial L}{\partial t}$ で与えられることを表している。

一方、[[粘性抵抗を無理矢理ラグランジアンで表すような場合>解析力学/ネーターの定理#ec972d15]]などでは、そもそも $L=T-U$ の形ではないラグランジアンを取る。そのような場合ハミルトニアンは系のエネルギーとは異なる量になるのであるが、その場合にも、

$$
\dot H=-\frac{\partial L}{\partial t}
$$

は系のエネルギー変化と関係が深いものになる。

* ハミルトニアンは座標によらない物理量である [#d447be4d]

ハミルトニアンの定義式に現れる $\displaystyle\sum_{k=1}^n p_k\dot q_k$ は以下に見るように座標変換に対して変化しない物理量である。

$$
\begin{aligned}
\sum_{i=1}^n p_i\dot q_i&=\sum_{i=1}^n \frac{\partial L}{\partial \dot q_i}\dot q_i\\
&=\sum_{i=1}^n
\sum_{j=1}^n \bigg\{\cancel\frac{\partial Q_j}{\partial \dot q_i} \frac{\partial L}{\partial Q_j}+\frac{\partial\dot Q_j}{\partial \dot q_i} \underbrace{\frac{\partial L}{\partial \dot Q_j}}_{P_j}\bigg\}
\sum_{k=1}^n\bigg\{\frac{\partial \dot q_i}{\partial Q_k} Q_k+\frac{\partial \dot q_i}{\partial \dot Q_k}\dot Q_k\bigg\}\\
&=\sum_{i=1}^n
\sum_{j=1}^n \frac{\partial Q_j}{\partial q_i} P_j
\sum_{k=1}^n\bigg\{\sum_{l=1}^n\frac{\partial^2 q_i}{\partial Q_l\partial Q_k} \dot Q_lQ_k+\frac{\partial q_i}{\partial Q_k}\dot Q_k\bigg\}\\
&=\sum_{j=1}^n\sum_{k=1}^n\bigg[
\sum_{l=1}^n\bigg\{\frac{\partial}{\partial Q_l}\sum_{i=1}^n \frac{\partial Q_j}{\partial q_i}\frac{\partial q_i}{\partial Q_k}\bigg\} \dot Q_l Q_k+
\bigg\{\sum_{i=1}^n \frac{\partial  Q_j}{\partial q_i}\frac{\partial q_i}{\partial Q_k}\bigg\} P_j\dot Q_k\bigg]\\
&=\sum_{j=1}^n\sum_{k=1}^n\bigg[
\sum_{l=1}^n\underbrace{\bigg(\frac{\partial}{\partial Q_l}\underbrace{\frac{\partial Q_j}{\partial Q_k}}_{\delta_{jk}}\bigg)}_{=\,0} \dot Q_l Q_k+
\underbrace{\frac{\partial  Q_j}{\partial Q_k}}_{\delta_{jk}} P_j\dot Q_k\bigg]\\
&=\sum_{j=1}^nP_j\dot Q_j\\
\end{aligned}
$$

すなわち異なる座標系 $Q_i$ に対してハミルトニアンを計算しても、
元の座標系 $q_i$ で計算した値と同じものが得られるのだ。

まあ、ハミルトニアンは系の全エネルギーを与えるような量なのだから
座標の取り方で変化するわけはなくて、上記の結果は当たり前と言えば当たり前なのだけれど。

~

>>> [[解析力学]] へ戻る

* 質問・コメント [#r1984971]

#article_kcaptcha
**ラグランジアンやハミルトニアンで変数を独立とすることについて [#wd533a4a]
>[[am]] (&timetag(2019-11-11T11:04:20+09:00, 2019-11-11 (月) 20:04:20);)~
~
初めまして。どうしても分からないことがあるので質問させて頂きます。~
L(q,qドット)やH(q,p)で、qとqドットあるいはqとpを独立変数とするという点が理解出来ません。~
例えば、エネルギー保存則を考えれば、qとqドットは当然独立ではありませんし、そもそもqドット=dq/dtを考えれば、常にqドットはqに従って変化していくものだと考えています。~
それなのに、Lでは独立変数としていることが理解出来ません。独立変数とはどういうことでしょうか?~
ご教授頂けますと幸いです。~

//
- 多くの方が躓く点かもしれませんね。[[最小点においては汎関数微分がゼロとなる>解析力学/ラグランジアン#ifcb74cb]] の説明をもう一度見ていただけるとよいと思うのですが、「作用(この積分の中では $\dot q=dq/dt$ で $\dot q$ は $q$ と独立に取れない)の $q$ による汎関数微分」をあくまで数学的な同値変形の結果として「ラグランジアンの $q,\dot q$ による偏微分(ここを考えているときは $q,\dot q$ は独立に取れるとする)で表すことができる」というのが話の流れになります。 -- [[武内(管理人)]] &new{2019-11-11 (月) 20:31:07};
- 「偏微分」は「複数ある変数のうち他を固定して1つだけを動かして微分するもの」ですが、ここで言う「$q$ による偏微分」は「$\dot q$ を固定して $q$ で微分したもの」である、というのが $q,\dot q$ を独立変数とした偏微分である、という話の意味になります。 -- [[武内(管理人)]] &new{2019-11-11 (月) 20:31:27};
- $\dot q=dq/dt$ なので独立には取れない、というのは正しい認識ですが、エネルギー保存則の話はちょっとずれてしまってますね。「実現するかどうか分からない(したがって、エネルギー保存則が成り立たなくても構わない)任意の経路のうち、作用積分を最小とするものが実際に起きる物理現象を表す(したがって、エネルギー保存則を満たす)」という話になるので、ここでエネルギーが保存しない経路も含めて考えることについては驚くところではないはずです。 -- [[武内(管理人)]] &new{2019-11-11 (月) 20:31:40};
- エネルギー保存に関しては理解できました。一つ目の•に関してですが、「数学的変形によってLのqとqドットによる偏微分で表せる。よって独立となる。」についてですが、本当は独立ではない(変分を考える際、qドットの増加量をdδq/dtとしているところがまさに独立ではないということ?)が、偏微分をする上では独立と見なせるということでよろしいですか?また、数学的変形というのはLの変分をテイラー展開した結果、Lの偏微分が出てくるということでよろしいですか?また、偏微分が出てくるのは分かりますが、わざわざ独立と考える意味はあるのでしょうか?長文になり、質問も分かりづらくて申し訳ありません。 -- [[am]] &new{2019-11-12 (火) 16:52:35};
- 元々は $q$ の変化に対する作用の変化を考えているのですが、$q$ を変えると $\dot q$ も変わることを考慮に入れて、求めるべき作用の変化を (1) $q$ が変わったことによる変化、と (2) $\dot q$ が変わったことによる変化、の和で表しているのです。(1) を考える際には仮に $\dot q$ が変わらないとしていて、(2) を考える際には仮に $q$ が変わらないとしている。わざわざ独立に考えているのは、そのように2つの成分に分けることで汎関数微分を $L$ の偏微分と結びつけることができて、有用な式変形が行えるから、と考えればいいと思います。どのように答えたら分かりやすいのか自信が持てないので、疑問が残る場合には質問を重ねて下さると私も勉強になります。 -- [[武内(管理人)]] &new{2019-11-12 (火) 18:07:11};
- 式変形としては、$L$ の全微分を $L$ の偏微分で表していることになりますが、テーラー展開の1次までを残していると考えても同じですね。$dL=\frac{\partial L}{\partial q}dq+\frac{\partial L}{\partial \dot q}d\dot q$ の形です。 -- [[武内(管理人)]] &new{2019-11-12 (火) 18:27:42};
- 自分なりに整理したことを項目に分けて確認させてください。①全微分dLはq、qドットが独立でなくても数学的に(∂L/∂q)δq+(∂L/∂qドット)δqドットと表せる。この式の意味は、qの変化δqと、それに伴うqドットの変化δqドットによるLの変化を、仮にδqとδqドットを独立に変化させると考えれば(∂L/∂q)δq+(∂L/∂qドット)δqドットと表せる。 -- [[am]] &new{2019-11-14 (木) 20:09:06};
- ②しかしδqドットはそもそもδqによって生じたものだから、その条件をどこかで考慮する必要がある。それは、全微分で表したのちに部分積分する際に、∂L/(∂qドット)δqドットを∂L/(∂qドット)dδq/dtと変形することで、本来独立でないという条件も式に入っている。 -- [[am]] &new{2019-11-14 (木) 20:10:48};
- ③L(q,qドット)でqとqドットが独立といっているのは、あくまで変分を求める際に独立になるということ。(最終的にはラグランジュ運動方程式においてqとqドットが独立になる。)ただし、変分を考えない段階、つまり単に作用を考える段階においては独立ではない。 -- [[am]] &new{2019-11-14 (木) 20:12:36};
- 以上のようにまとめてみたのですが、正しく理解できているでしょうか。 -- [[am]] &new{2019-11-14 (木) 20:14:42};
- はい、それでほぼ問題ないと思います。1点だけ、「ラグランジュ運動方程式において$q$と$\dot q$が独立になる」の部分に小さな違和感を覚えました。形式的に書かれたラグランジュ方程式に$L$の具体形を入れてニュートン方程式が現れることを追ってみて下さい。独立ってなんだ?となるんじゃないかと思います。ハミルトン方程式では$q$と$p$の関係が方程式自体から出てきます。「独立変数」の主な意味は偏微分する際に固定されるべき変数、という程度に捉えていいのではないでしょうか。 -- [[武内(管理人)]] &new{2019-11-15 (金) 11:58:04};
- もう一つ確認させて頂きたいのですが、「L(q,qドット)において、qとqドットは独立」という表現は間違いということでいいでしょうか?今までの話の流れだと間違いだと認識しているのですが、ネットや大学の先生の話では各時刻の位置と速度は独立あるいは初期条件では位置と速度は独立に取れるからqとqドットは独立だという人もいます。 -- [[am]] &new{2019-11-15 (金) 15:17:09};
- 間違っているかどうかは、「独立だから~~~である」というときの「~~~」の内容次第ですね。「$L(q,\dot q)$を考えるとき、$q$と$\dot q$は独立として扱う」が、$\partial L/\partial q$ を考えるときに $\dot q$ は固定して考える、という意味であれば間違いではありません。 -- [[武内(管理人)]] &new{2019-11-15 (金) 15:45:36};

#comment_kcaptcha


Counter: 44502 (from 2010/06/03), today: 9, yesterday: 0